Stronger than AM-GM and a conjectureQuestion about Rudin's Theorem 8.22 proof (Stirling's formula) - part...

What is the English pronunciation of "pain au chocolat"?

How could a planet have erratic days?

What is going on with 'gets(stdin)' on the site coderbyte?

Why should universal income be universal?

What does chmod -u do?

How do you make your own symbol when Detexify fails?

Why does a simple loop result in ASYNC_NETWORK_IO waits?

What should you do when eye contact makes your subordinate uncomfortable?

Extract more than nine arguments that occur periodically in a sentence to use in macros in order to typset

How do apertures which seem too large to physically fit work?

How much character growth crosses the line into breaking the character

Did arcade monitors have same pixel aspect ratio as TV sets?

What are some good ways to treat frozen vegetables such that they behave like fresh vegetables when stir frying them?

Are Captain Marvel's powers affected by Thanos' actions in Infinity War

Store Credit Card Information in Password Manager?

Is there a RAID 0 Equivalent for RAM?

Why does AES have exactly 10 rounds for a 128-bit key, 12 for 192 bits and 14 for a 256-bit key size?

Quoting Keynes in a lecture

Picking the different solutions to the time independent Schrodinger eqaution

Can the US President recognize Israel’s sovereignty over the Golan Heights for the USA or does that need an act of Congress?

How does a computer interpret real numbers?

Lowest total scrabble score

How can I write humor as character trait?

How to cover method return statement in Apex Class?



Stronger than AM-GM and a conjecture


Question about Rudin's Theorem 8.22 proof (Stirling's formula) - part (b)If $a_1a_2cdots a_n =1$, then $prod_{i=1}^n (1+a_i) geq 2^n$A conjecture similar to the Hardy inequalityA inequality about $x_1,x_2,ldots, x_n$Prove that for a family of $100n$ subsets of a $[n]$ such that each subset has size $m$, there exists two whose intersection is at least $m^2/10n$.Hard inequality with condition ($xyz=1$)Inequality using JensenRefinement of a strong inequalityPower sum inequalityA nice power sum inequalityA conjecture about power sum : $e^{ab}+e^{bc}+e^{ca}geq 3e^{sqrt{abc}}$ and $a+b+c=3$













1












$begingroup$



Let $a_i>0$ be $n$ numbers such that $prod_{i=1}^{n}a_ileq 1$ then we have:
$$sum_{i=1}^{n}a_igeq n left(prod_{i=1}^{n}a_i right)^{largeleft({n+sum_{i=1}^{n}a_i-n(prod_{i=1}^{n}a_i)^{1/n}}right)^{-1}}.$$




A possible way to generalize this is the following conjecture:




Let $f(x)$ be a increasing and convex function and $a_i$ be $n$ real numbers such that $f(a_i)>0$ with $prod_{i=1}^{n}f(a_i)leq 1$ then we have:
$$sum_{i=1}^{n}f(a_i)geq n left( prod_{i=1}^{n}f(a_i) right)^{large left(n+sum_{i=1}^{n}f(a_i)-nfleft(sum_{i=1}^{n}a_i/nright)right)^{-1}}.$$




Any hints would be appreciable to solve this. Thanks in advance.










share|cite|improve this question











$endgroup$








  • 1




    $begingroup$
    Where do these inequalities come from? Why do you think they are true? – And to which mathematical contest is the question related?
    $endgroup$
    – Martin R
    Feb 28 at 7:47






  • 6




    $begingroup$
    The first inequality is formulated as a fact. Could you provide a reference for those who (like me) are not familiar with that stronger version of the AM-GM inequality?
    $endgroup$
    – Martin R
    Feb 28 at 7:54










  • $begingroup$
    I have tried to improve the readability of your question by enlarging the indices. It is possible that I unintentionally changed the meaning of your question. Please proofread the question to ensure this has not happened.
    $endgroup$
    – GNUSupporter 8964民主女神 地下教會
    Feb 28 at 8:48










  • $begingroup$
    @GNUSupporter8964民主女神地下教會: It seems to be that your edit changed the expression in the second inequality. For example, it was $n+sum_{i=1}^{n}f(a_i)-nf(frac{sum_{i=1}^{n}a_i}{n})$ the exponent on the RHS.
    $endgroup$
    – Martin R
    Feb 28 at 10:13










  • $begingroup$
    @MartinR Nice catch! Nonetheless, this shows the importance of avoiding fractions in subscripts/superscripts. The letters were so small that I mistakenly thought that they were the same. There were old and experienced Math.SE users who can't read such small letters.
    $endgroup$
    – GNUSupporter 8964民主女神 地下教會
    Feb 28 at 10:25
















1












$begingroup$



Let $a_i>0$ be $n$ numbers such that $prod_{i=1}^{n}a_ileq 1$ then we have:
$$sum_{i=1}^{n}a_igeq n left(prod_{i=1}^{n}a_i right)^{largeleft({n+sum_{i=1}^{n}a_i-n(prod_{i=1}^{n}a_i)^{1/n}}right)^{-1}}.$$




A possible way to generalize this is the following conjecture:




Let $f(x)$ be a increasing and convex function and $a_i$ be $n$ real numbers such that $f(a_i)>0$ with $prod_{i=1}^{n}f(a_i)leq 1$ then we have:
$$sum_{i=1}^{n}f(a_i)geq n left( prod_{i=1}^{n}f(a_i) right)^{large left(n+sum_{i=1}^{n}f(a_i)-nfleft(sum_{i=1}^{n}a_i/nright)right)^{-1}}.$$




Any hints would be appreciable to solve this. Thanks in advance.










share|cite|improve this question











$endgroup$








  • 1




    $begingroup$
    Where do these inequalities come from? Why do you think they are true? – And to which mathematical contest is the question related?
    $endgroup$
    – Martin R
    Feb 28 at 7:47






  • 6




    $begingroup$
    The first inequality is formulated as a fact. Could you provide a reference for those who (like me) are not familiar with that stronger version of the AM-GM inequality?
    $endgroup$
    – Martin R
    Feb 28 at 7:54










  • $begingroup$
    I have tried to improve the readability of your question by enlarging the indices. It is possible that I unintentionally changed the meaning of your question. Please proofread the question to ensure this has not happened.
    $endgroup$
    – GNUSupporter 8964民主女神 地下教會
    Feb 28 at 8:48










  • $begingroup$
    @GNUSupporter8964民主女神地下教會: It seems to be that your edit changed the expression in the second inequality. For example, it was $n+sum_{i=1}^{n}f(a_i)-nf(frac{sum_{i=1}^{n}a_i}{n})$ the exponent on the RHS.
    $endgroup$
    – Martin R
    Feb 28 at 10:13










  • $begingroup$
    @MartinR Nice catch! Nonetheless, this shows the importance of avoiding fractions in subscripts/superscripts. The letters were so small that I mistakenly thought that they were the same. There were old and experienced Math.SE users who can't read such small letters.
    $endgroup$
    – GNUSupporter 8964民主女神 地下教會
    Feb 28 at 10:25














1












1








1





$begingroup$



Let $a_i>0$ be $n$ numbers such that $prod_{i=1}^{n}a_ileq 1$ then we have:
$$sum_{i=1}^{n}a_igeq n left(prod_{i=1}^{n}a_i right)^{largeleft({n+sum_{i=1}^{n}a_i-n(prod_{i=1}^{n}a_i)^{1/n}}right)^{-1}}.$$




A possible way to generalize this is the following conjecture:




Let $f(x)$ be a increasing and convex function and $a_i$ be $n$ real numbers such that $f(a_i)>0$ with $prod_{i=1}^{n}f(a_i)leq 1$ then we have:
$$sum_{i=1}^{n}f(a_i)geq n left( prod_{i=1}^{n}f(a_i) right)^{large left(n+sum_{i=1}^{n}f(a_i)-nfleft(sum_{i=1}^{n}a_i/nright)right)^{-1}}.$$




Any hints would be appreciable to solve this. Thanks in advance.










share|cite|improve this question











$endgroup$





Let $a_i>0$ be $n$ numbers such that $prod_{i=1}^{n}a_ileq 1$ then we have:
$$sum_{i=1}^{n}a_igeq n left(prod_{i=1}^{n}a_i right)^{largeleft({n+sum_{i=1}^{n}a_i-n(prod_{i=1}^{n}a_i)^{1/n}}right)^{-1}}.$$




A possible way to generalize this is the following conjecture:




Let $f(x)$ be a increasing and convex function and $a_i$ be $n$ real numbers such that $f(a_i)>0$ with $prod_{i=1}^{n}f(a_i)leq 1$ then we have:
$$sum_{i=1}^{n}f(a_i)geq n left( prod_{i=1}^{n}f(a_i) right)^{large left(n+sum_{i=1}^{n}f(a_i)-nfleft(sum_{i=1}^{n}a_i/nright)right)^{-1}}.$$




Any hints would be appreciable to solve this. Thanks in advance.







real-analysis inequality contest-math a.m.-g.m.-inequality






share|cite|improve this question















share|cite|improve this question













share|cite|improve this question




share|cite|improve this question








edited Feb 28 at 10:17









GNUSupporter 8964民主女神 地下教會

14k82650




14k82650










asked Feb 28 at 7:37









FatsWallersFatsWallers

1307




1307








  • 1




    $begingroup$
    Where do these inequalities come from? Why do you think they are true? – And to which mathematical contest is the question related?
    $endgroup$
    – Martin R
    Feb 28 at 7:47






  • 6




    $begingroup$
    The first inequality is formulated as a fact. Could you provide a reference for those who (like me) are not familiar with that stronger version of the AM-GM inequality?
    $endgroup$
    – Martin R
    Feb 28 at 7:54










  • $begingroup$
    I have tried to improve the readability of your question by enlarging the indices. It is possible that I unintentionally changed the meaning of your question. Please proofread the question to ensure this has not happened.
    $endgroup$
    – GNUSupporter 8964民主女神 地下教會
    Feb 28 at 8:48










  • $begingroup$
    @GNUSupporter8964民主女神地下教會: It seems to be that your edit changed the expression in the second inequality. For example, it was $n+sum_{i=1}^{n}f(a_i)-nf(frac{sum_{i=1}^{n}a_i}{n})$ the exponent on the RHS.
    $endgroup$
    – Martin R
    Feb 28 at 10:13










  • $begingroup$
    @MartinR Nice catch! Nonetheless, this shows the importance of avoiding fractions in subscripts/superscripts. The letters were so small that I mistakenly thought that they were the same. There were old and experienced Math.SE users who can't read such small letters.
    $endgroup$
    – GNUSupporter 8964民主女神 地下教會
    Feb 28 at 10:25














  • 1




    $begingroup$
    Where do these inequalities come from? Why do you think they are true? – And to which mathematical contest is the question related?
    $endgroup$
    – Martin R
    Feb 28 at 7:47






  • 6




    $begingroup$
    The first inequality is formulated as a fact. Could you provide a reference for those who (like me) are not familiar with that stronger version of the AM-GM inequality?
    $endgroup$
    – Martin R
    Feb 28 at 7:54










  • $begingroup$
    I have tried to improve the readability of your question by enlarging the indices. It is possible that I unintentionally changed the meaning of your question. Please proofread the question to ensure this has not happened.
    $endgroup$
    – GNUSupporter 8964民主女神 地下教會
    Feb 28 at 8:48










  • $begingroup$
    @GNUSupporter8964民主女神地下教會: It seems to be that your edit changed the expression in the second inequality. For example, it was $n+sum_{i=1}^{n}f(a_i)-nf(frac{sum_{i=1}^{n}a_i}{n})$ the exponent on the RHS.
    $endgroup$
    – Martin R
    Feb 28 at 10:13










  • $begingroup$
    @MartinR Nice catch! Nonetheless, this shows the importance of avoiding fractions in subscripts/superscripts. The letters were so small that I mistakenly thought that they were the same. There were old and experienced Math.SE users who can't read such small letters.
    $endgroup$
    – GNUSupporter 8964民主女神 地下教會
    Feb 28 at 10:25








1




1




$begingroup$
Where do these inequalities come from? Why do you think they are true? – And to which mathematical contest is the question related?
$endgroup$
– Martin R
Feb 28 at 7:47




$begingroup$
Where do these inequalities come from? Why do you think they are true? – And to which mathematical contest is the question related?
$endgroup$
– Martin R
Feb 28 at 7:47




6




6




$begingroup$
The first inequality is formulated as a fact. Could you provide a reference for those who (like me) are not familiar with that stronger version of the AM-GM inequality?
$endgroup$
– Martin R
Feb 28 at 7:54




$begingroup$
The first inequality is formulated as a fact. Could you provide a reference for those who (like me) are not familiar with that stronger version of the AM-GM inequality?
$endgroup$
– Martin R
Feb 28 at 7:54












$begingroup$
I have tried to improve the readability of your question by enlarging the indices. It is possible that I unintentionally changed the meaning of your question. Please proofread the question to ensure this has not happened.
$endgroup$
– GNUSupporter 8964民主女神 地下教會
Feb 28 at 8:48




$begingroup$
I have tried to improve the readability of your question by enlarging the indices. It is possible that I unintentionally changed the meaning of your question. Please proofread the question to ensure this has not happened.
$endgroup$
– GNUSupporter 8964民主女神 地下教會
Feb 28 at 8:48












$begingroup$
@GNUSupporter8964民主女神地下教會: It seems to be that your edit changed the expression in the second inequality. For example, it was $n+sum_{i=1}^{n}f(a_i)-nf(frac{sum_{i=1}^{n}a_i}{n})$ the exponent on the RHS.
$endgroup$
– Martin R
Feb 28 at 10:13




$begingroup$
@GNUSupporter8964民主女神地下教會: It seems to be that your edit changed the expression in the second inequality. For example, it was $n+sum_{i=1}^{n}f(a_i)-nf(frac{sum_{i=1}^{n}a_i}{n})$ the exponent on the RHS.
$endgroup$
– Martin R
Feb 28 at 10:13












$begingroup$
@MartinR Nice catch! Nonetheless, this shows the importance of avoiding fractions in subscripts/superscripts. The letters were so small that I mistakenly thought that they were the same. There were old and experienced Math.SE users who can't read such small letters.
$endgroup$
– GNUSupporter 8964民主女神 地下教會
Feb 28 at 10:25




$begingroup$
@MartinR Nice catch! Nonetheless, this shows the importance of avoiding fractions in subscripts/superscripts. The letters were so small that I mistakenly thought that they were the same. There were old and experienced Math.SE users who can't read such small letters.
$endgroup$
– GNUSupporter 8964民主女神 地下教會
Feb 28 at 10:25










1 Answer
1






active

oldest

votes


















4





+50







$begingroup$

For the first inequality, write $A=tfrac1n sum a_i$ and $G=prod a_i^{1/n}.$ We need to prove $A^{1+A-G}geq G.$
But $$A^{A-G}=(A^A)^{(A-G)/A}geq e^{-(A-G)/A}geq 1-(A-G)/A=G/A$$
where the middle inequality uses $A^Ageq e^{-e^{-1}}geq e^{-1}.$



The generalization is false, if my calculations are correct. Take
begin{align*}
n&=4\
a_1&=tfrac12\
a_i&=1qquad(2leq ileq 4)\
f(x)&=begin{cases}x/e^2& (xleq 7/8)\
7/8e^2 + 8(1/e-7/8e^2)(x-7/8)& (xgeq 7/8)
end{cases}\
f(a_1)&=1/2e^2\
f(a_i)&=1/eqquad(2leq ileq 4)\
A&=tfrac1nsum f(a_i)approx0.29283\
G&=prod f(a_i)^{1/n}=(1/2e)^{1/4}/eapprox0.24092\
F&=f(sum a_i/n)=f(7/8)=7/8e^2approx 0.11842.
end{align*}

Then $0.80718approx A^{A-F}<G/Aapprox 0.82274,$ but the desired inequality is equivalent to $A^{A-F}geq G/A.$






share|cite|improve this answer











$endgroup$













    Your Answer





    StackExchange.ifUsing("editor", function () {
    return StackExchange.using("mathjaxEditing", function () {
    StackExchange.MarkdownEditor.creationCallbacks.add(function (editor, postfix) {
    StackExchange.mathjaxEditing.prepareWmdForMathJax(editor, postfix, [["$", "$"], ["\\(","\\)"]]);
    });
    });
    }, "mathjax-editing");

    StackExchange.ready(function() {
    var channelOptions = {
    tags: "".split(" "),
    id: "69"
    };
    initTagRenderer("".split(" "), "".split(" "), channelOptions);

    StackExchange.using("externalEditor", function() {
    // Have to fire editor after snippets, if snippets enabled
    if (StackExchange.settings.snippets.snippetsEnabled) {
    StackExchange.using("snippets", function() {
    createEditor();
    });
    }
    else {
    createEditor();
    }
    });

    function createEditor() {
    StackExchange.prepareEditor({
    heartbeatType: 'answer',
    autoActivateHeartbeat: false,
    convertImagesToLinks: true,
    noModals: true,
    showLowRepImageUploadWarning: true,
    reputationToPostImages: 10,
    bindNavPrevention: true,
    postfix: "",
    imageUploader: {
    brandingHtml: "Powered by u003ca class="icon-imgur-white" href="https://imgur.com/"u003eu003c/au003e",
    contentPolicyHtml: "User contributions licensed under u003ca href="https://creativecommons.org/licenses/by-sa/3.0/"u003ecc by-sa 3.0 with attribution requiredu003c/au003e u003ca href="https://stackoverflow.com/legal/content-policy"u003e(content policy)u003c/au003e",
    allowUrls: true
    },
    noCode: true, onDemand: true,
    discardSelector: ".discard-answer"
    ,immediatelyShowMarkdownHelp:true
    });


    }
    });














    draft saved

    draft discarded


















    StackExchange.ready(
    function () {
    StackExchange.openid.initPostLogin('.new-post-login', 'https%3a%2f%2fmath.stackexchange.com%2fquestions%2f3129884%2fstronger-than-am-gm-and-a-conjecture%23new-answer', 'question_page');
    }
    );

    Post as a guest















    Required, but never shown

























    1 Answer
    1






    active

    oldest

    votes








    1 Answer
    1






    active

    oldest

    votes









    active

    oldest

    votes






    active

    oldest

    votes









    4





    +50







    $begingroup$

    For the first inequality, write $A=tfrac1n sum a_i$ and $G=prod a_i^{1/n}.$ We need to prove $A^{1+A-G}geq G.$
    But $$A^{A-G}=(A^A)^{(A-G)/A}geq e^{-(A-G)/A}geq 1-(A-G)/A=G/A$$
    where the middle inequality uses $A^Ageq e^{-e^{-1}}geq e^{-1}.$



    The generalization is false, if my calculations are correct. Take
    begin{align*}
    n&=4\
    a_1&=tfrac12\
    a_i&=1qquad(2leq ileq 4)\
    f(x)&=begin{cases}x/e^2& (xleq 7/8)\
    7/8e^2 + 8(1/e-7/8e^2)(x-7/8)& (xgeq 7/8)
    end{cases}\
    f(a_1)&=1/2e^2\
    f(a_i)&=1/eqquad(2leq ileq 4)\
    A&=tfrac1nsum f(a_i)approx0.29283\
    G&=prod f(a_i)^{1/n}=(1/2e)^{1/4}/eapprox0.24092\
    F&=f(sum a_i/n)=f(7/8)=7/8e^2approx 0.11842.
    end{align*}

    Then $0.80718approx A^{A-F}<G/Aapprox 0.82274,$ but the desired inequality is equivalent to $A^{A-F}geq G/A.$






    share|cite|improve this answer











    $endgroup$


















      4





      +50







      $begingroup$

      For the first inequality, write $A=tfrac1n sum a_i$ and $G=prod a_i^{1/n}.$ We need to prove $A^{1+A-G}geq G.$
      But $$A^{A-G}=(A^A)^{(A-G)/A}geq e^{-(A-G)/A}geq 1-(A-G)/A=G/A$$
      where the middle inequality uses $A^Ageq e^{-e^{-1}}geq e^{-1}.$



      The generalization is false, if my calculations are correct. Take
      begin{align*}
      n&=4\
      a_1&=tfrac12\
      a_i&=1qquad(2leq ileq 4)\
      f(x)&=begin{cases}x/e^2& (xleq 7/8)\
      7/8e^2 + 8(1/e-7/8e^2)(x-7/8)& (xgeq 7/8)
      end{cases}\
      f(a_1)&=1/2e^2\
      f(a_i)&=1/eqquad(2leq ileq 4)\
      A&=tfrac1nsum f(a_i)approx0.29283\
      G&=prod f(a_i)^{1/n}=(1/2e)^{1/4}/eapprox0.24092\
      F&=f(sum a_i/n)=f(7/8)=7/8e^2approx 0.11842.
      end{align*}

      Then $0.80718approx A^{A-F}<G/Aapprox 0.82274,$ but the desired inequality is equivalent to $A^{A-F}geq G/A.$






      share|cite|improve this answer











      $endgroup$
















        4





        +50







        4





        +50



        4




        +50



        $begingroup$

        For the first inequality, write $A=tfrac1n sum a_i$ and $G=prod a_i^{1/n}.$ We need to prove $A^{1+A-G}geq G.$
        But $$A^{A-G}=(A^A)^{(A-G)/A}geq e^{-(A-G)/A}geq 1-(A-G)/A=G/A$$
        where the middle inequality uses $A^Ageq e^{-e^{-1}}geq e^{-1}.$



        The generalization is false, if my calculations are correct. Take
        begin{align*}
        n&=4\
        a_1&=tfrac12\
        a_i&=1qquad(2leq ileq 4)\
        f(x)&=begin{cases}x/e^2& (xleq 7/8)\
        7/8e^2 + 8(1/e-7/8e^2)(x-7/8)& (xgeq 7/8)
        end{cases}\
        f(a_1)&=1/2e^2\
        f(a_i)&=1/eqquad(2leq ileq 4)\
        A&=tfrac1nsum f(a_i)approx0.29283\
        G&=prod f(a_i)^{1/n}=(1/2e)^{1/4}/eapprox0.24092\
        F&=f(sum a_i/n)=f(7/8)=7/8e^2approx 0.11842.
        end{align*}

        Then $0.80718approx A^{A-F}<G/Aapprox 0.82274,$ but the desired inequality is equivalent to $A^{A-F}geq G/A.$






        share|cite|improve this answer











        $endgroup$



        For the first inequality, write $A=tfrac1n sum a_i$ and $G=prod a_i^{1/n}.$ We need to prove $A^{1+A-G}geq G.$
        But $$A^{A-G}=(A^A)^{(A-G)/A}geq e^{-(A-G)/A}geq 1-(A-G)/A=G/A$$
        where the middle inequality uses $A^Ageq e^{-e^{-1}}geq e^{-1}.$



        The generalization is false, if my calculations are correct. Take
        begin{align*}
        n&=4\
        a_1&=tfrac12\
        a_i&=1qquad(2leq ileq 4)\
        f(x)&=begin{cases}x/e^2& (xleq 7/8)\
        7/8e^2 + 8(1/e-7/8e^2)(x-7/8)& (xgeq 7/8)
        end{cases}\
        f(a_1)&=1/2e^2\
        f(a_i)&=1/eqquad(2leq ileq 4)\
        A&=tfrac1nsum f(a_i)approx0.29283\
        G&=prod f(a_i)^{1/n}=(1/2e)^{1/4}/eapprox0.24092\
        F&=f(sum a_i/n)=f(7/8)=7/8e^2approx 0.11842.
        end{align*}

        Then $0.80718approx A^{A-F}<G/Aapprox 0.82274,$ but the desired inequality is equivalent to $A^{A-F}geq G/A.$







        share|cite|improve this answer














        share|cite|improve this answer



        share|cite|improve this answer








        edited Mar 15 at 10:27

























        answered Mar 15 at 10:16









        DapDap

        19k842




        19k842






























            draft saved

            draft discarded




















































            Thanks for contributing an answer to Mathematics Stack Exchange!


            • Please be sure to answer the question. Provide details and share your research!

            But avoid



            • Asking for help, clarification, or responding to other answers.

            • Making statements based on opinion; back them up with references or personal experience.


            Use MathJax to format equations. MathJax reference.


            To learn more, see our tips on writing great answers.




            draft saved


            draft discarded














            StackExchange.ready(
            function () {
            StackExchange.openid.initPostLogin('.new-post-login', 'https%3a%2f%2fmath.stackexchange.com%2fquestions%2f3129884%2fstronger-than-am-gm-and-a-conjecture%23new-answer', 'question_page');
            }
            );

            Post as a guest















            Required, but never shown





















































            Required, but never shown














            Required, but never shown












            Required, but never shown







            Required, but never shown

































            Required, but never shown














            Required, but never shown












            Required, but never shown







            Required, but never shown







            Popular posts from this blog

            Magento 2 - Add success message with knockout Planned maintenance scheduled April 23, 2019 at 23:30 UTC (7:30pm US/Eastern) Announcing the arrival of Valued Associate #679: Cesar Manara Unicorn Meta Zoo #1: Why another podcast?Success / Error message on ajax request$.widget is not a function when loading a homepage after add custom jQuery on custom themeHow can bind jQuery to current document in Magento 2 When template load by ajaxRedirect page using plugin in Magento 2Magento 2 - Update quantity and totals of cart page without page reload?Magento 2: Quote data not loaded on knockout checkoutMagento 2 : I need to change add to cart success message after adding product into cart through pluginMagento 2.2.5 How to add additional products to cart from new checkout step?Magento 2 Add error/success message with knockoutCan't validate Post Code on checkout page

            Fil:Tokke komm.svg

            Where did Arya get these scars? Unicorn Meta Zoo #1: Why another podcast? Announcing the arrival of Valued Associate #679: Cesar Manara Favourite questions and answers from the 1st quarter of 2019Why did Arya refuse to end it?Has the pronunciation of Arya Stark's name changed?Has Arya forgiven people?Why did Arya Stark lose her vision?Why can Arya still use the faces?Has the Narrow Sea become narrower?Does Arya Stark know how to make poisons outside of the House of Black and White?Why did Nymeria leave Arya?Why did Arya not kill the Lannister soldiers she encountered in the Riverlands?What is the current canonical age of Sansa, Bran and Arya Stark?